Question and Answers Forum

All Questions      Topic List

Integration Questions

Previous in All Question      Next in All Question      

Previous in Integration      Next in Integration      

Question Number 122349 by liberty last updated on 16/Nov/20

 Find the polynomial P(x) of least degree  that has a maximum equal to 6 at x=1  and minimum equal to 2 at x=3.

$$\:\mathrm{Find}\:\mathrm{the}\:\mathrm{polynomial}\:\mathrm{P}\left(\mathrm{x}\right)\:\mathrm{of}\:\mathrm{least}\:\mathrm{degree} \\ $$$$\mathrm{that}\:\mathrm{has}\:\mathrm{a}\:\mathrm{maximum}\:\mathrm{equal}\:\mathrm{to}\:\mathrm{6}\:\mathrm{at}\:\mathrm{x}=\mathrm{1} \\ $$$$\mathrm{and}\:\mathrm{minimum}\:\mathrm{equal}\:\mathrm{to}\:\mathrm{2}\:\mathrm{at}\:\mathrm{x}=\mathrm{3}.\: \\ $$

Commented by benjo_mathlover last updated on 17/Nov/20

my answer :   the polynomial of the least degree  with roots x_1 =1 and x_2 =3 has the   form a(x−1)(x−3). Hence   P ′(x)=a(x−1)(x−3)=a(x^2 −4x+3)  Since at the point x=1 there must be  P(1)=6 , we have   P(x)=∫_1 ^x P ′(x) dx + 6   P(x)= a∫_1 ^x (x^2 −4x+3)dx+6   P(x)= a((x^3 /3)−2x^2 +3x−(4/3))+6.  The coefficient a is determined  from the condition P(3)=2, whence  a =3 , Hence P(x)=x^3 −6x^2 +9x+2.

$${my}\:{answer}\::\: \\ $$$${the}\:{polynomial}\:{of}\:{the}\:{least}\:{degree} \\ $$$${with}\:{roots}\:{x}_{\mathrm{1}} =\mathrm{1}\:{and}\:{x}_{\mathrm{2}} =\mathrm{3}\:{has}\:{the}\: \\ $$$${form}\:{a}\left({x}−\mathrm{1}\right)\left({x}−\mathrm{3}\right).\:{Hence}\: \\ $$$${P}\:'\left({x}\right)={a}\left({x}−\mathrm{1}\right)\left({x}−\mathrm{3}\right)={a}\left({x}^{\mathrm{2}} −\mathrm{4}{x}+\mathrm{3}\right) \\ $$$${Since}\:{at}\:{the}\:{point}\:{x}=\mathrm{1}\:{there}\:{must}\:{be} \\ $$$${P}\left(\mathrm{1}\right)=\mathrm{6}\:,\:{we}\:{have}\: \\ $$$${P}\left({x}\right)=\underset{\mathrm{1}} {\overset{{x}} {\int}}{P}\:'\left({x}\right)\:{dx}\:+\:\mathrm{6}\: \\ $$$${P}\left({x}\right)=\:{a}\underset{\mathrm{1}} {\overset{{x}} {\int}}\left({x}^{\mathrm{2}} −\mathrm{4}{x}+\mathrm{3}\right){dx}+\mathrm{6}\: \\ $$$${P}\left({x}\right)=\:{a}\left(\frac{{x}^{\mathrm{3}} }{\mathrm{3}}−\mathrm{2}{x}^{\mathrm{2}} +\mathrm{3}{x}−\frac{\mathrm{4}}{\mathrm{3}}\right)+\mathrm{6}. \\ $$$${The}\:{coefficient}\:{a}\:{is}\:{determined} \\ $$$${from}\:{the}\:{condition}\:{P}\left(\mathrm{3}\right)=\mathrm{2},\:{whence} \\ $$$${a}\:=\mathrm{3}\:,\:{Hence}\:{P}\left({x}\right)={x}^{\mathrm{3}} −\mathrm{6}{x}^{\mathrm{2}} +\mathrm{9}{x}+\mathrm{2}. \\ $$

Answered by mr W last updated on 17/Nov/20

f′(x)=k(x−1)(x−3)=k(x^2 −4x+3)  f(x)=k((x^3 /3)−2x^2 +3x+c)  f(1)=k((4/3)+c)=6  f(3)=kc=2  ⇒k=3  ⇒c=(2/3)  ⇒f(x)=x^3 −6x^2 +9x+2

$${f}'\left({x}\right)={k}\left({x}−\mathrm{1}\right)\left({x}−\mathrm{3}\right)={k}\left({x}^{\mathrm{2}} −\mathrm{4}{x}+\mathrm{3}\right) \\ $$$${f}\left({x}\right)={k}\left(\frac{{x}^{\mathrm{3}} }{\mathrm{3}}−\mathrm{2}{x}^{\mathrm{2}} +\mathrm{3}{x}+{c}\right) \\ $$$${f}\left(\mathrm{1}\right)={k}\left(\frac{\mathrm{4}}{\mathrm{3}}+{c}\right)=\mathrm{6} \\ $$$${f}\left(\mathrm{3}\right)={kc}=\mathrm{2} \\ $$$$\Rightarrow{k}=\mathrm{3} \\ $$$$\Rightarrow{c}=\frac{\mathrm{2}}{\mathrm{3}} \\ $$$$\Rightarrow{f}\left({x}\right)={x}^{\mathrm{3}} −\mathrm{6}{x}^{\mathrm{2}} +\mathrm{9}{x}+\mathrm{2} \\ $$

Commented by mr W last updated on 16/Nov/20

Commented by liberty last updated on 16/Nov/20

yes...thanks...

$$\mathrm{yes}...\mathrm{thanks}... \\ $$

Commented by benjo_mathlover last updated on 17/Nov/20

typo sir. we get k=3 . it should  be f(x)=3((x^3 /3)−2x^2 +3x+(2/3))    f(x) = x^3 −6x^3 +9x+2

$${typo}\:{sir}.\:{we}\:{get}\:{k}=\mathrm{3}\:.\:{it}\:{should} \\ $$$${be}\:{f}\left({x}\right)=\mathrm{3}\left(\frac{{x}^{\mathrm{3}} }{\mathrm{3}}−\mathrm{2}{x}^{\mathrm{2}} +\mathrm{3}{x}+\frac{\mathrm{2}}{\mathrm{3}}\right) \\ $$$$\:\:{f}\left({x}\right)\:=\:{x}^{\mathrm{3}} −\mathrm{6}{x}^{\mathrm{3}} +\mathrm{9}{x}+\mathrm{2} \\ $$

Commented by mr W last updated on 17/Nov/20

yes, thanks!

$${yes},\:{thanks}! \\ $$

Terms of Service

Privacy Policy

Contact: info@tinkutara.com